Forma canónica de Jordan

Proporcionamos ejercicios sobre la forma canónica de Jordan.

RESUMEN TEÓRICO
  • Teorema.  Sea $E$ espacio vectorial sobre el cuerpo $\mathbb{K}$ de dimensión finita $n$ y $f:E\to E$ un endomorfismo. Supongamos que los polinomios característico y mínimo de $f$ son respectivamente: $$\begin{aligned}&\chi (\lambda)=(\lambda-\lambda_1)^{n_1}(\lambda-\lambda_2)^{n_2}\ldots (\lambda-\lambda_p)^{n_p},\\
    &\mu (\lambda)=(\lambda-\lambda_1)^{m_1}(\lambda-\lambda_2)^{m_2}\ldots (\lambda-\lambda_p)^{m_p},\end{aligned}$$ en donde los $\lambda_i$ son escalares distintos dos a dos. Entonces, existe una base $B_J$ de $E$ respecto de la cual la matriz $J$ de $f$ es diagonal por bloques, siendo cada uno de los bloques de $J$ de la forma:  $$J_{ij}=\begin{bmatrix} \lambda_i & 1 & 0 & \ldots & 0 & 0\\ 0 &\lambda_i & 1 &\ldots & 0 & 0 \\ \vdots&&&&&\vdots \\ 0 & 0 & 0 & \ldots & \lambda_i & 1\\ 0 & 0 & 0 & \ldots & 0 & \lambda_i\end{bmatrix}.$$ Para cada valor propio $\lambda_i$ se verifica:
    $(i)\;$ Existe al menos un $J_{ij}$ de dimensión $m_i,$ los demas $J_{ij}$ son de órdenes $\leq m_i.$
    $(ii)\;$ La suma de los órdenes de los $J_{ij}$ es $n_i.$
    $(iii)\;$ El número de bloques $J_{ij}$ coincide con la multiplicidad geométrica de $\lambda_i$
  • Definición.  A la matriz $J$ se la denomina forma canónica de Jordan de $f$ y a la base $B_J,$ base de Jordan para $f.$
    Enunciado
  1. Hallar las posibles formas de Jordan para un endomorfismo cuyos polinomio característico y mínimo son respectivamente: $$\chi(\lambda)=(\lambda-2)^4(\lambda-3)^3,\quad \mu(\lambda)=(\lambda-2)^2(\lambda-3)^3.$$
  2. Hallar las posibles formas de Jordan para un endomorfismo cuyos polinomio característico y mínimo son respectivamente: $$\chi(\lambda)=(\lambda-7)^5,\quad \mu(\lambda)=(\lambda-7)^2.$$
  3. Hallar las posibles formas de Jordan para un endomorfismo cuyos polinomio característico y mínimo son respectivamente: $$\chi(\lambda)=(\lambda-a)^3(\lambda-b)^2,\quad \mu(\lambda)=(\lambda-a)(\lambda-b),\quad (a\neq b).$$
  4. Hallar el polinomio mínimo y la forma canónica de Jordan de la matriz $$A=\begin{bmatrix}{0}&{0}&{0} & 0\\{2}&{0}&{0} & 0\\{2}&{2}&{0} & 0\\ 2 & 2 & 0 & 0 \end{bmatrix}.$$
  5. Sea $\;M=\begin{bmatrix}{b}&{0}&{0}&b+3\\{0}&{0}&{0}&\;\;b\\{b}&{0}&{0}&-2\\{0}&{0}&{0}&\;\;b\end{bmatrix}\in\mathbb{R}^{4\times 4}.$
    Determinar su forma canónica de Jordan según los valores del parámetro $b.$
    Solución
  1. Las posibles formas canónicas de Jordan de $f$ son: $$J=\begin{bmatrix}{2}&{1}&{}&&&&\\{0}&{2}&{}&&&&\\{}&{}&{2}&1&&&\\{}&{}&{0}&2&&&
    \\{}&{}&{}&&3&1&0
    \\{}&{}&{}&&0&3&1
    \\{}&{}&{}&&0&0&3\end{bmatrix}\;(\text{si }\dim V_2=2).$$ $$J=\begin{bmatrix}{2}&{1}&{}&&&&\\{0}&{2}&{}&&&&\\{}&{}&{2}&&&&\\{}&{}&{}&2&&&
    \\{}&{}&{}&&3&1&0
    \\{}&{}&{}&&0&3&1
    \\{}&{}&{}&&0&0&3\end{bmatrix}\;(\text{si }\dim V_2=3).$$
  2. Tenemos:$$J=\begin{bmatrix}{7}&{1}&{}&&\\
    {0}&{7}&{}&&\\
    {}&{}&{7}&1&\\
    {}&{}&{0}&7&\\
    {}&{}&{}&&7\end{bmatrix}\;(\text{si }\dim V_7=3).$$ $$J=\begin{bmatrix}{7}&{1}&{}&&\\
    {0}&{7}&{}&&\\
    {}&{}&{7}&&\\
    {}&{}&{}&7&\\
    {}&{}&{}&&7\end{bmatrix}\;(\text{si }\dim V_7=4).$$
  3. Necesariamente ha de ser $$J=\begin{bmatrix}{a}&{}&{}&&\\
    {}&{a}&{}&&\\
    {}&{}&{a}&&\\
    {}&{}&{}&b&\\
    {}&{}&{}&&b\end{bmatrix},$$ por tanto el endomorfismo es diagonalizable.
  4. Polinomio característico de $A:$ $$\begin{vmatrix}{-\lambda}&{0}&{0} & 0\\{2}&{-\lambda}&{0} & 0\\{2}&{2}&{-\lambda} & 0\\ 2 & 2 & 0 & -\lambda \end{vmatrix}=(-\lambda)^4=\lambda^4.$$ El polinomio mínimo sabemos que divide al característico y tiene los mismos factores irreducibles, por tanto los posibles polinomios mínimos de $A$ son: $$\mu_1(\lambda)=\lambda,\;\; \mu_2(\lambda)=\lambda^2,\;\; \mu_3(\lambda)=\lambda^3,\;\; \mu_4(\lambda)=\lambda^4.$$ Tenemos $$\mu_1(A)=A\neq 0,\;\;\mu_2(A)=A^2=\begin{bmatrix}{0}&{0}&{0} & 0\\{0}&{0}&{0} & 0\\{4}&{0}&{0} & 0\\ 4 & 0 & 0 & 0 \end{bmatrix}\neq 0,\;\;\mu_3(A)=A^3=0.$$ En consecuencia, el polinomio mínimo de es $\mu_3(\lambda)=\lambda^3,$ y la forma canónica de Jordan de $A:$ $$J=\begin{bmatrix}{0}&{1}&{0} & \\{0}&{0}&{1} & \\{0}&{0}&{0} & \\ & & & 0 \end{bmatrix}.$$
  5. Valores propios de $M:$ $$\begin{vmatrix}{b-\lambda}&{0}&{0}&b+3\\{0}&{-\lambda}&{0}&\;\;b\\{b}&{0}&{-\lambda}&-2\\{0}&{0}&{0}&\;\;b-\lambda\end{vmatrix}=(b-\lambda)^2(\lambda)^2=0\Leftrightarrow \lambda=b\vee \lambda=0,$$ valores propios que son al menos dobles.
    Primer caso. Si $b=0$ el único valor propio es $\lambda=0$ (cuádruple) y $$\dim V_0=4-\text{rg }(M-0I)=4-\text{rg }\begin{bmatrix}{0}&{0}&{0}&\;\;3\\{0}&{0}&{0}&\;\;0\\{0}&{0}&{0}&-2\\{0}&{0}&{0}&\;\;0\end{bmatrix}=4-1=3.$$ El número de cajas es tres, por tanto la forma de Jordan de $M$ es $$J=\begin{bmatrix}{0}&{1}&{}&\\{0}&{0}&{}&\\{}&{}&{0}&\\{}&{}&{}&0\end{bmatrix}.$$ Segundo caso. Si $b\neq 0$ los valores propios son $\lambda=b$ y $\lambda=0$ (dobles) y $$\dim V_b=4-\text{rg }(M-bI)=4-\text{rg }\begin{bmatrix}{0}&{0}&{0}&b+3\\{0}&{-b}&{0}&b\\{b}&{0}&{-b}&-2\\{0}&{0}&{0}& 0\end{bmatrix}$$ $$=4-\text{rg }\begin{bmatrix}{b}&{0}&{-b}&-2\\{0}&{-b}&{0}&b\\{0}&{0}&{0}&b+3\end{bmatrix}\underbrace{=}_{b\neq 0}\left \{ \begin{matrix} 4-3=1& \mbox{ si }& b\neq -3 \\4-2=2 & \mbox{ si }& b=-3,\end{matrix}\right.$$ $$\dim V_0=4-\text{rg }(M-0I)=4-\text{rg }\begin{bmatrix}{b}&{0}&{0}&b+3\\{0}&{0}&{0}&\;\;b\\{b}&{0}&{0}&-2\\{0}&{0}&{0}&\;\;b\end{bmatrix}$$ $$=4-\text{rg }\begin{bmatrix}{b}&b+3\\{0}&\;\;b\\{b}&-2\\{0}&\;\;b\end{bmatrix}\underbrace{=}_{b\neq 0}4-2=2.$$ Como la dimensión proporciona el número de cajas asociadas al correspondiente valor propio, queda $$J=\begin{bmatrix}{b}&{1}&{}&\\{0}&{b}&{}&\\{}&{}&{0}&\\{}&{}&{}&0\end{bmatrix}\text{ si }b\neq -3,\quad J=\begin{bmatrix}{-3}&{}&{}&\\{}&{-3}&{}&\\{}&{}&{0}&\\{}&{}&{}&0\end{bmatrix}\text{ si }b\neq 3.$$
Esta entrada ha sido publicada en Álgebra y etiquetada como , , . Guarda el enlace permanente.